Re: [obm-l]

2021-04-23 Por tôpico Marcos Martinelli
> > On Fri, Apr 23, 2021 at 4:43 PM Marcos Martinelli > wrote: > >> Opa, pessoal. Pensei nos últimos dias no problema seguinte. Cheguei a uma >> solução um pouco mais genérica, mas me deu trabalho. Gostaria de estudar >> outras abordagens. >> >> Problema)

[obm-l]

2021-04-23 Por tôpico Marcos Martinelli
Opa, pessoal. Pensei nos últimos dias no problema seguinte. Cheguei a uma solução um pouco mais genérica, mas me deu trabalho. Gostaria de estudar outras abordagens. Problema) Prove que raiz (2) + raiz_cúbica (2) é irracional. Na sequência posto um rascunho do que pensei. Obrigado. -- Esta

[obm-l] Re: [obm-l] Polinômio

2020-10-26 Por tôpico Marcos Martinelli
Correção: fazendo y=1/(r+i). Em seg, 26 de out de 2020 às 10:49, Marcos Martinelli < mffmartine...@gmail.com> escreveu: > Sendo i a unidade imaginária: > > 1/(1+r_k^2) = 1/(2i)*(1/(r_k-i)-1/(r_k+i) (k=[1,n], onde r_k <> {-i,i}). > > i) Seja z_k = 1/(r_k-i) e fazendo

[obm-l] Re: [obm-l] Polinômio

2020-10-26 Por tôpico Marcos Martinelli
0i+21)/(-7i+2) (II). Usando (I) e (II): Soma_(k=[1,n]) 1/(1+r_k^2) = 1/(2i)*(-(-20i+21)/(7i+2) -(-(20i+21)/(-7i+2))) = 1/(2i)*1/(-49)*(-80i-294i) = 187/49. Em dom., 25 de out. de 2020 às 10:25, Marcos Martinelli < mffmartine...@gmail.com> escreveu: > Sendo i o complexo imaginário: > &g

[obm-l] Re: [obm-l] Polinômio

2020-10-25 Por tôpico Marcos Martinelli
Correção: 1/(1+r_k^2) = 1/(2i)*(1/(r_k-i)-1/(r_k+i) Em dom, 25 de out de 2020 às 10:25, Marcos Martinelli < mffmartine...@gmail.com> escreveu: > Sendo i o complexo imaginário: > > 1/(1+r_k^2) = 1/(2i)*(1/(r_k-i)+1/(r_k+i) > > Depois você deve considerar dois novos polinô

[obm-l] Re: [obm-l] Polinômio

2020-10-25 Por tôpico Marcos Martinelli
Sendo i o complexo imaginário: 1/(1+r_k^2) = 1/(2i)*(1/(r_k-i)+1/(r_k+i) Depois você deve considerar dois novos polinômios com as seguintes mudanças de variáveis: . x=1/y-i . x=1/y+i Devemos então calcular as somas dos inversos das raizes nesses dois polinômios para termos como calcular o

[obm-l] Re: [obm-l] Teorema do valor intermediário

2016-05-03 Por tôpico Marcos Martinelli
Precisamos supor que f é contínua. Considere g: (0,1/2) -> R tal que g(x) = f(x + 1/2) - f(x) para todo x em (0, 1/2). Se f(1/2) = f(0), é satisfeito o enunciado. Vamos supor, então, f(1/2) <> f(0). Como g é contínua, g(0) = f(1/2) - f(0) e g(1/2) = f(1) - f(1/2) = - (f(1/2) - f(0)) = - g(0),

[obm-l] Re: [obm-l] Congruéncias de 1^k + 2^k .... + (p - 1)^k

2015-05-05 Por tôpico Marcos Martinelli
Pequena correção no enunciado: Sejam k, p naturais, sendo p um primo diferente de 2. -- Esta mensagem foi verificada pelo sistema de antiv�rus e acredita-se estar livre de perigo.

[obm-l] Re: [obm-l] Congruéncias de 1^k + 2^k .... + (p - 1)^k

2015-05-05 Por tôpico Marcos Martinelli
Acredito que devemos ter p primo e diferente de 2 também. Reformulando o enunciado então: Sejam k, p naturais, sendo p um primo. Provar que: i) se k == 0 (mod p - 1) = soma{ t = 1 }_{ p - 1 } t^k == - 1 (mod p); e ii) se k 0 (mod p - 1) = soma{ t = 1 }_{ p - 1 } t^k == 0 (mod p). Solução:

[obm-l] Re: [obm-l] Re: [obm-l] Congruéncias de 1^k + 2^k .... + (p - 1)^k

2015-05-05 Por tôpico Marcos Martinelli
É verdade. Minha demonstração foi para um caso particular mesmo. Já conhecia a versão desse problema para p primo diferente de 2... :) ! Mais tarde vou tentar estudar a sua demonstração. Obrigado. Em 5 de maio de 2015 14:00, Pedro José petroc...@gmail.com escreveu: Boa tarde! Marcos

Re: [obm-l] Problema de Geometria

2015-04-30 Por tôpico Marcos Martinelli
Não é uma solução com geometria pura. ___ Lema) Um quadrilátero XYZW é inscritível se somente se XZ * YW = XY * ZW + XW * YZ . Solução) Sejam p, r, a, b e c a notação usual de um triângulo ABC qualquer. Seja F o ponto de

[obm-l] Re: Soma de Quadrados

2014-12-19 Por tôpico Marcos Martinelli
Na realidade, o pedido do problema é: calcular lim P_N quando N - + infty. -- Esta mensagem foi verificada pelo sistema de antiv�rus e acredita-se estar livre de perigo.

Re: [obm-l] Soma de Quadrados

2014-12-19 Por tôpico Marcos Martinelli
Marcos Martinelli mffmartine...@gmail.com: Um número natural m é chamado interessante se existirem n e k naturais tais que n k 0, k é ímpar e ainda: m = n^2 - (n - 1)^2 + (n - 2)^2 - ... - (n - k)^2 . Seja P_N a probabilidade de escolhermos um número interessante dentre os primeiros N naturais

Re: [obm-l] Teorema

2014-12-19 Por tôpico Marcos Martinelli
Sim. Complicada. Decorre de um teorema de Chebyshev. 2014-12-19 17:32 GMT-02:00 marcone augusto araújo borges marconeborge...@hotmail.com: Seja p um primo.Existe um primo p´tal que p p´ 2p. A demostração é complicada?Onde achar? -- Esta mensagem foi verificada pelo sistema de antivírus e

Re: [obm-l] Quadrados perfeitos

2014-12-18 Por tôpico Marcos Martinelli
Os casos 0! e 1! são os únicos exemplos em que um fatorial pode ser um quadrado perfeito. Vamos considerar N = 2. Seja {p_i} (i natural) a sequência dos primos. Vamos usar a seguinte desigualdade (Chebychev): p_(n+1) 2 * p_(n) para todo n natural. Seja também j natural tal que p_(j) = N

[obm-l] Soma de Quadrados

2014-12-18 Por tôpico Marcos Martinelli
Um número natural m é chamado interessante se existirem n e k naturais tais que n k 0, k é ímpar e ainda: m = n^2 - (n - 1)^2 + (n - 2)^2 - ... - (n - k)^2 . Seja P_N a probabilidade de escolhermos um número interessante dentre os primeiros N naturais. Calcular lim (P_N / N) quando N - +

Re: [obm-l] Desigualdade

2014-04-30 Por tôpico Marcos Martinelli
Considerando x,y,z 0: Faça a = y/x, b = z/x e c = x/z (repare que abc = 1). x/(x+y) + y/(y+z) + z/(z+x) = 1/(1+a) + 1/(1+b) + 1/(1+c) = (3 + 2(a+b+c) + (ab+ac+bc)) / (1 + (a+b+c) + (ab+ac+bc) + abc). Nessa última expressão: S1 = a+b+c, S2 = ab+ac+bc. Lembrando que abc = 1, vamos ter o

[obm-l] Re: [obm-l] Somatório

2014-04-17 Por tôpico Marcos Martinelli
Esse link é interessante: https://www.youtube.com/watch?v=0Oazb7IWzbA Em 12 de abril de 2014 12:53, Vanderlei Nemitz vanderma...@gmail.comescreveu: Pessoal, vi em um site a seguinte camiseta: http://www.zazzle.com.br/teoria_da_corda-235032240070858893 Lembrei que uma vez um aluno meu

[obm-l] soma da Eureka

2013-12-31 Por tôpico Marcos Martinelli
-- Date: Mon, 30 Dec 2013 20:34:20 -0200 Subject: Re: [obm-l] soma da Eureka From: mffmartine...@gmail.com To: obm-l@mat.puc-rio.br Na linha seguinte: * {1/2 . sum{k = 2}^{100} [-1/k + 1/(k - 1)]} Em segunda-feira, 30 de dezembro de 2013, Marcos Martinelli escreveu: Uma pequena correção na

Re: [obm-l] soma da Eureka

2013-12-30 Por tôpico Marcos Martinelli
A gente pode considerar f(k) = (k + 1)/(k^2 + k + 1). Podemos mostrar a seguinte relação: 1/(k^4 + k^2 + 1) = 1/2 . [(k + 1)/(k^2 + k + 1) - (k - 1)/(k^2 - k +1)] = 1/2 . [f(k) - f(k - 1) + 1/(k^2 - k +1)] . Assim, a soma que queremos é tal que: sum{k = 1}^{100} 1/(k^4 + k^2 + 1) = [1/2 . sum{k

Re: [obm-l] soma da Eureka

2013-12-30 Por tôpico Marcos Martinelli
Uma pequena correção na escrita (quinta linha): * = 1/2 . f(100) +1/2 . sum{k = 2}^{100} 1/(k^2 - k + 1) Em segunda-feira, 30 de dezembro de 2013, Marcos Martinelli escreveu: A gente pode considerar f(k) = (k + 1)/(k^2 + k + 1). Podemos mostrar a seguinte relação: 1/(k^4 + k^2 + 1) = 1/2

Re: [obm-l] soma da Eureka

2013-12-29 Por tôpico Marcos Martinelli
Para resolver o problema proposto, repare que: f(x) + f(1 - x) = 2/(4^x + 2) + 2/[4^(1 - x) + 2] = 2/(4^x + 2) + 4^x/(2 + 4^x) = 1. Em domingo, 29 de dezembro de 2013, Luís escreveu: Sauda,c~oes, Adaptando o problema 3 da p. 37 da Eureka 37, existiria ?? uma forma fechada para a soma S(n)

Re: [obm-l] soma da Eureka

2013-12-29 Por tôpico Marcos Martinelli
f(x) + f(1 - x) = a^x/(a^x + sqr(a)) + a^(1 - x)/[a^(1 - x) + sqr(a)] = a^x/(a^x + sqr(a)) + a/(a + a^x . sqr(a)) = a^x/(a^x + sqr(a)) + sqr(a)/(a^x + sqr(a)) = 1. Em domingo, 29 de dezembro de 2013, Luís escreveu: Oi, oi Marcos, Verdade. O problema 4 tem uma solução parecida: f(x) + f(1/x)

[obm-l] Re: [obm-l] Recorreção - IME 2014

2013-12-11 Por tôpico Marcos Martinelli
Mostre sua solução?! Em 11 de dezembro de 2013 14:36, João Maldonado joao_maldona...@hotmail.com escreveu: Boa Tarde pra todo mundo :) Eu prestei o IME no mês de outubro e recentemente chegou a prova corrigida no meu email, Eu fiquei com nota 9 em matemática, mas jurava que tinha acertado

[obm-l] Re: [obm-l] àlgebra linear

2013-09-02 Por tôpico Marcos Martinelli
Podemos supor que as matrizes A, B e C pertencem todas a C_n (são matrizes de elementos complexos e de ordem n). Vamos considerar três casos: i) A = B e det(A) 0 - C = 2A^2 - C . (A^(-1)/sqr(2))^(2) = 2 . A^2 . (A^(-1))^(2) . 1/2 = (A . A^(-1))^(2) = I^(2) = I, o que mostra que C é

Re: [obm-l] Matriz positiva definida

2013-08-08 Por tôpico Marcos Martinelli
Seja A pertencente a M_n (R) (A é uma matriz do espaço das matrizes quadradas de ordem n *com coeficientes reais*). *Lema 01)* Se A é simétrica - todos seus autovalores são números reais. *obs* (corolário do Lema 01): dado que temos todos os autovalores reais, sempre podemos escolher os

[obm-l] Re: [obm-l] Valor máximo

2013-08-04 Por tôpico Marcos Martinelli
Como ele é do terceiro grau, vai ter que cruzar o eixo dos x pelo menos uma vez. No ponto de máximo ele tangência o eixo dos x, não o cruza. Por isso tem uma raiz dupla no ponto de tangência. Em sábado, 3 de agosto de 2013, marcone augusto araújo borges escreveu: Eu não entendi ´´esse polinomio

[obm-l] Re: [obm-l] RE: [obm-l] Re: [obm-l] Valor máximo

2013-08-02 Por tôpico Marcos Martinelli
Eu, de fato, não demonstrei nada... só quis justificar uma abordagem para desvendar o mistério da inequação que eu propus. Por isso que falei No rascunho. Para esse caso, como existe o máximo absoluto entre - 1 e + 1, a abordagem funcionou. Daí é só fazer a volta, com a inequação. Espero ter

[obm-l] Re: [obm-l] Valor máximo

2013-08-01 Por tôpico Marcos Martinelli
No rascunho, você pode tentar fazer o seguinte: vamos admitir que g(t) = - 2t^3 + 2t possui um máximo M. Esse máximo deve ser positivo já que g(t) = 2t . (1 - t^2) é positivo para 0 t 1. Agora você pode definir o seguinte polinômio h(t) = - 2t^3 + 2t - M. Essa função de terceiro grau deve tocar

Re: [obm-l] Provar: |x^n| = |x|^n

2013-07-31 Por tôpico Marcos Martinelli
Considere os seguintes casos: i) x = 0 - x^n = 0 - |x^n| = x^n = |x|^n; ii) x 0 (considere x = - y (onde y 0). Temos: |x| = - x = y) e n = 2k (onde k é natural) - x^n = (-y)^(2k) = y^(2k) 0 - |x^n| = x^n = y^(2k) = |x|^n; iii) x 0 (considere x = - y (onde y 0). Temos: |x| = - x = y) e n =

[obm-l] Re: [obm-l] Fwd: Equação Trigonométrica

2013-07-31 Por tôpico Marcos Martinelli
cos x = 0 - x = k.pi + pi/2. Se você colocar 2k.pi + pi/2 só vai estar contando os arcos côngruos a pi/2. Vai esquecer os côngruos a 3.pi/2. cos x = sqrt(2)/2 - x = 2kpi +- pi/4. cos x = - sqrt(2)/2 - x = 2kpi +- 3.pi/4. Em 31 de julho de 2013 06:04, Rafael Dumas dk.virtua...@gmail.comescreveu:

[obm-l] Re: [obm-l] Valor máximo

2013-07-31 Por tôpico Marcos Martinelli
f(x) = sex(x) . sex(2x) = sen(x) . [2sen(x)cos(x)] = 2cos(x)sen^2(x) = 2cos(x).(1-cos^2(x)). Fazendo cox(x) = t (- 1 = t = + 1), devemos descobrir o máximo da seguinte função: g(t) = - 2t^3 + 2t. Sabemos que para t = - 1, temos: t + 2sqrt(3)/3 = 0 (- 2sqrt(3)/3 - 1) - (t - sqrt(3)/3)^2 . (t +

Re: [obm-l] |(-x)^n)| = |x^n| (Como provar?)

2013-07-31 Por tôpico Marcos Martinelli
i) se n = 2k (para k natural) - (-x)^n = (-1)^(2k) . x^(2k) = x^n - |(-x)^n| = |x^n|. ii) se n = 2k + 1 (para k natural) - (-x)^n = (-1)^(2k+1) . x^(2k+1) = - x^n - |(-x)^n| = |- x^n| = |x^n|. Essa última igualdade ocorre pois |-a| = |a| para qualquer a real. Em 31 de julho de 2013 20:22, Pedro

[obm-l] Re: [obm-l] Re: [obm-l] Re: [obm-l] Re: [obm-l] Re: [obm-l] Re: [obm-l] Re: [obm-l] ajuda em exercício de geometria

2013-07-30 Por tôpico Marcos Martinelli
2BD=2DE*=EC* Consegue fazer a construção agora? =D Em 27 de julho de 2013 11:54, Marcos Martinelli mffmartine...@gmail.comescreveu: Acho que dá pra provar que não existem pontos D e E pertencentes à BC e que satisfaçam as outras condições do enunciado. i) supondo que D e E pertencem ao

[obm-l] Re: [obm-l] questão de GA simples

2013-07-29 Por tôpico Marcos Martinelli
Se um plano é ortogonal a um vetor de coordenadas (a,b,c) então a equação desse plano é: ax + by + cz = d, onde d é uma constante. Em 29 de julho de 2013 23:47, Hermann ilhadepaqu...@bol.com.br escreveu: ** Meus amigos, não estou enxergando: Determinar a equação cartesiana e paramétrica do

[obm-l] Re: [obm-l] Re: [obm-l] Re: [obm-l] Re: [obm-l] Re: [obm-l] ajuda em exercício de geometria

2013-07-27 Por tôpico Marcos Martinelli
. Em 26 de julho de 2013 20:19, Bruno Rodrigues brunorodrigues@gmail.comescreveu: Pelo que eu entendi da questão,sim. Saudações Em 26 de julho de 2013 17:00, Marcos Martinelli mffmartine...@gmail.comescreveu: Então o problema está dizendo que os segmentos de reta BD, DE e EC são

[obm-l] Re: [obm-l] Sistema de equações trigonométricas e exponenciais

2013-07-26 Por tôpico Marcos Martinelli
Da segunda equação, devemos ter: x 0 e y 0 (*). Suponhamos, sem perda de generalidade, que x = 0 - e^x = 1 - e^y = (1 - e^x) = 0. Absurdo, pois e^y 0 para qualquer y real. I) sen (x + y) = sen(x) + sen(y) - sen (x + y) - sen(x) = sen(y) - 2 . sen(y/2) . cos(x + y/2) = 2 . sen(y/2) . cos(y/2).

[obm-l] Re: [obm-l] RE: [obm-l] Desigualdade das médias

2013-07-26 Por tôpico Marcos Martinelli
Lema 1) x + y + z = raiz (3 . (xy + yz + zx)) para quaisquer x, y e z positivos. Prova: Sabemos que: (x - y)^2 + (y - z)^2 + (z - x)^2 = 0 [*a igualdade ocorre se somente se x=y=z*]. Desenvolvendo, teremos: 2.(x^2 + y^2 + z^2) - 2xy - 2yz - 2zx = 0 - x^2 + y^2 + z^2 + 2xy + 2yz + 2zx= 3xy + 3yz

[obm-l] Re: [obm-l] ajuda em exercício de geometria

2013-07-26 Por tôpico Marcos Martinelli
Tem certeza dessa condição: 2BD=2DE=2EC? Achei meio estranho colocar o fator dois em todos os membros. Em 24 de julho de 2013 21:25, Bruno Rodrigues brunorodrigues@gmail.comescreveu: Oi pessoal,será que alguém consegue me dar uma luz nessa questão de geometria? Seja ABC um

[obm-l] Re: [obm-l] Re: [obm-l] Re: [obm-l] ajuda em exercício de geometria

2013-07-26 Por tôpico Marcos Martinelli
julho de 2013 14:12, Marcos Martinelli mffmartine...@gmail.comescreveu: Tem certeza dessa condição: 2BD=2DE=2EC? Achei meio estranho colocar o fator dois em todos os membros. Em 24 de julho de 2013 21:25, Bruno Rodrigues brunorodrigues@gmail.com escreveu: Oi pessoal,será que alguém

[obm-l] Re: [obm-l] Re: [obm-l] Sistema de equações trigonométricas e exponenciais

2013-07-26 Por tôpico Marcos Martinelli
, às 13:21, Marcos Martinelli mffmartine...@gmail.com escreveu: Da segunda equação, devemos ter: x 0 e y 0 (*). Suponhamos, sem perda de generalidade, que x = 0 - e^x = 1 - e^y = (1 - e^x) = 0. Absurdo, pois e^y 0 para qualquer y real. I) sen (x + y) = sen(x) + sen(y) - sen (x + y) - sen(x

[obm-l] Re: [obm-l] É possível que um polinômio e uma exponencial coincidam ?

2013-07-26 Por tôpico Marcos Martinelli
Acho que não poderia ocorrer concordância em uma infinidade de pontos. Considere g tal que g(x) = exp(x) - P(x), onde P(x) seria nosso polinômio. Por hipótese, existiriam infinitos pontos pertencentes a [a,b] tais que a função g é nula nesse intervalo. Como g é de classe c^{+ oo} nesse

[obm-l] Re: [obm-l] Polinômio curioso

2013-07-24 Por tôpico Marcos Martinelli
Seja G um polinômio de grau (n+1) tal que G(x) = x . P(x) - 1 (*) para qualquer x real. Fazendo x = k (k natural tal que 1 = k = n + 1), obteremos G(k) = 0 para todos os (n + 1) k´s. Portanto, temos todas as raízes de G e podemos escrever: G(x) = A . produtório (1 = k = n + 1) (x - k).

[obm-l] Re: [obm-l] Equação do terceiro grau

2013-07-24 Por tôpico Marcos Martinelli
Primeiro, faça a seguinte mudança de variáveis: x = z - 1. Substituindo na equação do terceiro grau, teremos: (z^3 - 3z^2 +3z -1) + 3(z^2 - 2z + 1) - 2(z - 1) + 1 = 0 - z^3 - 5z + 5 = 0 (*). Para descobrir a raiz irracional, podemos fazer uma nova mudança de variáveis. Queremos encontrar p e q

[obm-l] Re: [obm-l] Re: [obm-l] Re: [obm-l] Equação do terceiro grau

2013-07-24 Por tôpico Marcos Martinelli
imaginárias, não podem ser obtidas? Em 24 de julho de 2013 13:57, Marcos Martinelli mffmartine...@gmail.comescreveu: Primeiro, faça a seguinte mudança de variáveis: x = z - 1. Substituindo na equação do terceiro grau, teremos: (z^3 - 3z^2 +3z -1) + 3(z^2 - 2z + 1) - 2(z - 1) + 1 = 0 - z

Re: [obm-l] x^n = y^n = |x| = |y|

2013-07-17 Por tôpico Marcos Martinelli
Se x^n = y^n - |x^n| = |y^n| - |x|^n = |y|^n (|ab| = |a| . |b| para quaisquer a,b reais) - |x|^n - |y|^n = 0. Podemos supor, por absurdo, que: |x| |y|. Assim, podemos dividir e multiplicar o lado esquerdo de (*) por (|x| - |y|). Teremos: (|x| - |y|) . (|x|^n - |y|^n)/(|x| - |y|) = 0 - (|x| -

Re: [obm-l] Primos

2013-07-12 Por tôpico Marcos Martinelli
Ponce 2013/7/12 Marcos Martinelli mffmartine...@gmail.com Acho que não existe uma fórmula fechada para os primos. Acho que tentamos encontrá-la há um bom tempo... mas sem sucesso, apesar de inúmeras outras portas que foram abertas com a teoria analítica dos números. Em sexta-feira, 12 de

[obm-l] Re: [obm-l] Re: [obm-l] Análise combinatória

2013-07-12 Por tôpico Marcos Martinelli
Só não entendi essa parte: 100-(2+2+2+1)=97. Em 12 de julho de 2013 09:08, Marcos Martinelli mffmartine...@gmail.comescreveu: Legal. Em 12 de julho de 2013 09:02, Rogerio Ponce abrlw...@gmail.com escreveu: Ola' Artur, como queremos que a distancia minima entre os elementos seja de pelo

[obm-l] Re: [obm-l] Análise combinatória

2013-07-11 Por tôpico Marcos Martinelli
Seja {A_n} a quantidade de seqüências com 4 números escolhidos de 1 a n tais que a diferença positiva seja maior ou igual a 2 (n=4). Seja {B_n} a quantidade de seqüências com 3 números escolhidos de 1 a n tais que a diferença positiva seja maior ou igual a 2 (n=3). Seja {C_n} a quantidade de

Re: [obm-l] Primos

2013-07-11 Por tôpico Marcos Martinelli
Acho que não existe uma fórmula fechada para os primos. Acho que tentamos encontrá-la há um bom tempo... mas sem sucesso, apesar de inúmeras outras portas que foram abertas com a teoria analítica dos números. Em sexta-feira, 12 de julho de 2013, Nehab escreveu: Oi, Marcone, Números primos

Re: [obm-l] soma de quadrados

2012-06-17 Por tôpico Marcos Martinelli
(5^2 + 9^2).(12^2 + 17^2) = 60^2 + 85^2 + 108^2 + 153^2 = (60 + 153)^2 - 2.60.153 + 108^2 + 85^2 = 213^2 + (108^2 - 2.60.153 + 85^2) = 213^2 + (108 - 85)^2 = 213^2 + 23^2. Resposta: 213 + 23 = 236. Letra e). Em 17 de junho de 2012 15:44, Vanderlei * vanderma...@gmail.com escreveu: Se (5^2 +

Re: [obm-l] Ajuda Divisibilidade

2012-06-17 Por tôpico Marcos Martinelli
Se 7 | a + 3b - a + 3b = 7q, onde q é algum inteiro. Assim: 13.(a + 3b) = 7.13.q - 13a + 39 b = 7.13.q - 13a + 11b = 7.13.q - 28b = 7. (13q - 4b) - 7 | 13a + 11b. Em 17 de junho de 2012 16:54, Thiago Bersch thiago_t...@hotmail.comescreveu: 1°: Mostre que se 7 | a + 3b então 7| 13a + 11b,

[obm-l] Re: [obm-l] RES: [obm-l] Re: [obm-l] Re: [obm-l] Re: [obm-l] Re: [obm-l] Re: [obm-l] Re: [obm-l] Re: [obm-l] Re: [obm-l] heptágono regular

2012-03-29 Por tôpico Marcos Martinelli
Pois é, galera. A discussão foi muito interessante, e a solução geométrica muito legal. Mas não podemos negar que a questão é bastante difícil... acho difícil haver uma outra solução puramente geométrica. Enfim, o Colégio Naval é uma prova bastante interessante e, normalmente, os gabaritos que

[obm-l] Re: [obm-l] Re: [obm-l] Re: [obm-l] Re: [obm-l] Re: [obm-l] heptágono regular

2012-03-27 Por tôpico Marcos Martinelli
Peço desculpas por ter sido muito formal nesta questão. É que, pra mim, realmente não foi tão intuitivo supor que o perímetro seria crescente. Deve haver sim uma solução por geometria pura, mas ficarei devendo. Agora, quanto à questão levantada pelo último email, posso contribuir um pouco:

[obm-l] Re: [obm-l] Re: [obm-l] Re: [obm-l] heptágono regular

2012-03-26 Por tôpico Marcos Martinelli
Não sei se provar que o perímetro do hexágono é menor que o do heptágono é assim *tão banal*. Deu trabalho pra eu demonstrar isso (como feito em email anterior). Mas, com certeza, deve ter um jeito bem mais simples do que as funções que analisei. Em 26 de março de 2012 10:26, Carlos Nehab

[obm-l] Re: [obm-l] RES: [obm-l] Re: [obm-l] Re: [obm-l] Re: [obm-l] heptágono regular

2012-03-26 Por tôpico Marcos Martinelli
perímetro máximo: o perímetro do próprio círculo (2 pi r). ** ** Albert Bouskela bousk...@msn.com ** ** *De:* owner-ob...@mat.puc-rio.br [mailto:owner-ob...@mat.puc-rio.br] *Em nome de *Marcos Martinelli *Enviada em:* 26 de março de 2012 14:06 *Para:* obm-l@mat.puc-rio.br

[obm-l] Re: [obm-l] Re: [obm-l] Re: [obm-l] heptágono regular

2012-03-25 Por tôpico Marcos Martinelli
O perímetro deste heptágono pode ser calculado ao olharmos para os triângulos isósceles formados pelo centro do círculo e por vértices adjacentes do heptágono. Assim: 2p_(heptágono) = (2 * 2.5 * sen(pi/7)) * 7 = 35 * sen(pi/7) (*) Lema 01) Seja g: A = [ 0,pi/6 ] --- R tal que g(x) = x * cos(x)

Re: [obm-l] Desigualdade fatorial

2012-03-25 Por tôpico Marcos Martinelli
Bernardo, creio que, ao considerar as tangentes, podemos melhorar sim as desigualdades. Tentei incrementar um pouco mais minha solução e demonstrei as seguintes desigualdades: n! = n^n (***) * raiz(n) / (e^((2*n^2-3*n+1)/(4*n))) = (**) n^n / (e^((2*n^2-3*n+1)/(2*n))) = (*) n^n / (e^(n-1)), para

Re: [obm-l] Desigualdade fatorial

2012-03-25 Por tôpico Marcos Martinelli
Pequena correção: n! = *(***)* n^n * raiz(n) / (e^((2*n^2-3*n+1)/(4*n))) = *(**)* n^n / (e^((2*n^2-3*n+1)/(2*n))) = *(*)* n^n / (e^(n-1)), Os parênteses seguidos de asterisco procurar identificar as desigualdades citadas no email anterior.

Re: [obm-l] Desigualdade fatorial

2012-03-25 Por tôpico Marcos Martinelli
Fala, Bernardo. Existe um pequeno erro sim no meu denominador. Mas vou tentar esboçar aqui as contas: i) pelos trapézios (considerando n = 2): sum_{k=2}^{n} 1/2 . [(ln(t) -1/t) + ln(t)] int_{1}^{n} ln(t) . dt. Após algumas contas, chegamos à seguinte expressão: ln(n!) n . ln(n) - n + 1 + 1/2 .

Re: [obm-l] Desigualdade fatorial

2012-03-23 Por tôpico Marcos Martinelli
Uma desigualdade um pouco mais forte (e com uma demonstração legal) seria a seguinte: (n!) = (n^n)/(e^((2*n^2-3*n+1)/(2*n))) Em 23 de março de 2012 15:21, Bernardo Freitas Paulo da Costa bernardo...@gmail.com escreveu: 2012/3/23 terence thirteen peterdirich...@gmail.com: Em 22 de março de

Re: [obm-l] Desigualdade fatorial

2012-03-23 Por tôpico Marcos Martinelli
: Basta provar que (1+1/n)^n=3 para todo n (e não será necessário falar em limites). De fato, isto é equivalente a 3n^n=(n+1)^n, que é equivalente a (n+1).(n/3)^n=((n+1)/3)^(n+1)**, e agora é usar o PIF. A. Citando Marcos Martinelli mffmartine...@gmail.com: Uma

Re: [obm-l] EDO

2010-04-01 Por tôpico Marcos Martinelli
Creio que você errou no delta da equação do segundo grau. Teríamos: x1 = [1-sqrt(- 4h+1)]/2. x2 = [1+sqrt(- 4h+1)]/2. E aqui devemos considerar também o caso em que delta é negativo (h 1/4)! = Instruções para entrar na

Re: [obm-l] numero irracional

2010-03-23 Por tôpico Marcos Martinelli
Tal número é na verdade natural e igual a 1. = Instruções para entrar na lista, sair da lista e usar a lista em http://www.mat.puc-rio.br/~obmlistas/obm-l.html

Re: [obm-l] ajuda

2009-11-11 Por tôpico Marcos Martinelli
Na minha solução, você consegue calcular a soma para uma parcela finita de termos. Agora, como w = e^(ix) = cos(x) + isen(x) tem módulo unitário, para as duas pg´s convergirem devemos ter |a| 1. = Instruções para entrar na

Re: [obm-l] ajuda

2009-11-10 Por tôpico Marcos Martinelli
Repare que cos (kx) = [e^(ikx) + e^(-ikx)]/2 = (w^k + 1/w^k)/2, onde w = e^(ix). Assim nosso somatório será: 1 + soma(1 = k = n) [(aw)^k + (a/w)^k]/2. Repare agora que este somatório é na realidade a soma de duas PG's. Espero ter ajudado.

[obm-l] Re: [obm-l] Re: [obm-l] Re: [obm-l] recorrência

2009-06-19 Por tôpico Marcos Martinelli
É o seguinte: quando temos uma recorrência linaer homogênea a menos de uma constante, podemos sempre chutar uma outra recorrência {t_n} tal que s_n = t_n + k. Se substituirmos na equação recorrente, encontramos k. No nosso caso, k deve ser - 3.

[obm-l] Re: [obm-l] recorrência

2009-06-18 Por tôpico Marcos Martinelli
Considere {t(n)} (n natural) tal que s(n) = t(n) - 3. Substituindo na recorrência, teremos: t(n) - 3 = 2[t(n - 1) - 3] + 3 - t(n) = 2t(n - 1). A solução geral para esta recorrência é claramente t(n) = t(1)*2^(n - 1). Como t(1) = s(1) + 3 = 4, teremos t(n) = 4*2^(n - 1) = 2^(n + 1). Logo

Re: [obm-l] [OFF] perseguicao

2009-01-22 Por tôpico Marcos Martinelli
CADA UMA QUE AQUI APARECE... = Instruções para entrar na lista, sair da lista e usar a lista em http://www.mat.puc-rio.br/~obmlistas/obm-l.html =

Re: [obm-l] Questão ESaF de função

2008-08-25 Por tôpico Marcos Martinelli
É isso aí. Essa questão é do ISS NATAL, prova recente da ESAF. O examinador não foi feliz no enunciado. A questão teria de ser anulada. = Instruções para entrar na lista, sair da lista e usar a lista em

Re: [obm-l] Dúvida

2008-07-22 Por tôpico Marcos Martinelli
O correto seria S = (a^m - 1)/a-1 x (b^n - 1)/b-1 x (c^p - 1)/c-1. = Instruções para entrar na lista, sair da lista e usar a lista em http://www.mat.puc-rio.br/~obmlistas/obm-l.html

Re: [obm-l] Matrizes

2007-07-02 Por tôpico Marcos Martinelli
Basta observar que detX0 - X é inversível. = Instruções para entrar na lista, sair da lista e usar a lista em http://www.mat.puc-rio.br/~nicolau/olimp/obm-l.html

Re: [obm-l]

2007-04-26 Por tôpico Marcos Martinelli
Se x-0^+, naturalmente x0. Como 2/x-1[2/x]=2/x - 2/3-x/3x/3*[2/x]=2/3. Pelo Teorema do Confronto temos, portanto, lim(x-0^+)(x/3.[2/x])=2/3. Mas no outro item, basta observar que 2/x.[x/3]=0 para qq 0x1. Logo o limite procurado é nulo.

Re: [obm-l] Determinante

2006-03-27 Por tôpico Marcos Martinelli
A é anti-simétrica - A = -A^(t) - det[A] = det[-A^(t)] - det[A] = (-1)^{n}det[A]. Mas como n é ímpar, temos: det[A] = -det[A] - det[A]=0. c.q.d = Instruções para entrar na lista, sair da lista e usar a lista em

Re: [obm-l] questões de olim internacional

2005-12-14 Por tôpico Marcos Martinelli
Legal!

Re: [obm-l] DESIGUALDADE

2005-12-14 Por tôpico Marcos Martinelli
Faça a seguinte mudança de variáveis a=px, b=py e c=pz, onde p é o semiperímetro do triângulo e agora teremos que mostrar que 2x^2(1-x)+2y^2(1-y)+2z^2(1-z)=3xyz - 2(x^2+y^2+z^2)-2(x^3+y^3+z^3)=3xyz - 2[(x+y+z)^2-2xy-2xz-2yz]-2(x^3+y^3+z^3)+6xyz=9xyz - 2[4-2xy-2xz-2yz]-2[x^3+y^3+z^3-3xyz]=9xyz -

[obm-l] Questão legal

2005-12-13 Por tôpico Marcos Martinelli
Seja and sequence Considere também Calcular onde n como de usual tende para +infinito.

Re: [obm-l] questões de olim internacional

2005-12-13 Por tôpico Marcos Martinelli
Na questão 74, faça y=x^2-3x-2 e obtenha o seguinte sistema de equações: .y=x^2-3x-2 .x=y^2-3y-2 E agora subtraia as duas equações.

Re: [obm-l] Questão

2005-12-12 Por tôpico Marcos Martinelli
Se a+b+c=0 - a+b=-c - a^2+2ab+b^2=c^2. Como a^2+b^2=1-c^2 - 1-c^2+2ab=c^2 - c^2=(1+2ab)/2. Mas (a^2+b^2)^2=(1-c^2)^2 - a^4+b^4+2a^2b^2=[(1-2ab)/2]^2 - a^4+b^4+2(ab)^2=[1-4ab+4(ab)^2]/4 - a^4+b^4+2(ab)^2=1/4-ab+(ab)^2. Mas ainda temos que ab=c^2-1/2 - a^4+b^4+(c^2-1/2)^2=1/4-(c^2-1/2). E então

Re: [obm-l] analise limite

2005-12-12 Por tôpico Marcos Martinelli
{[n^(1/n)] - 1}^n=e^{ln{[n^(1/n)] - 1}^n}}=e^{n*ln{[n^(1/n)] - 1}}. É fácil de demonstrar que lim n^[1/n]=1. Logo o expoente tende pra -infinito e o termo todo tende pra zero.

Re: [obm-l] analise limite

2005-12-12 Por tôpico Marcos Martinelli
É simples. Como disse lim [n^1/n]=1 - lim[n^1/n-1]=0 - lim{ln[n^1/n-1]}=-infinito. Como lim n=+infinito, podemos concluir que: lim{n*ln[n^1/n-1]}=-infinito.

Re: [obm-l] Sequencia

2005-12-12 Por tôpico Marcos Martinelli
Vou provar o caso 1). O caso 2) seria análogo. lim{a_n/b_n}=0 - Para qualquer L0, existe N natural tal que para todo n natural tal que nN então |a_n/b_n|L. Podemos concluir que |a_k/b_k|L para todo k natural tal que Nk=n e então podemos escrever -La_k/b_kL - -L*b_ka_kL*b_k -

Re: [obm-l] Sequencias e series

2005-12-11 Por tôpico Marcos Martinelli
Na verdade, S seria o limite de (p^(n)-1)/(p-1). E a sequência {a_n} é na verdade uma P.G.

[obm-l] Mais uma legal

2005-12-06 Por tôpico Marcos Martinelli
Seja n natural maior ou igual que 2. Provar que 1/(n+1)*Somatório(1=k=n)[1/(2*k-1)]1/n*Somatório(1=k=n)[1/(2*k)]. Um abraço!

Re: [obm-l] Teorema de Cayley-Hamilton

2005-12-06 Por tôpico Marcos Martinelli
Será que tem como você enviar para o meu e-mail também? Obrigado!!!

Re: [obm-l] sem ser com calculadora

2005-12-05 Por tôpico Marcos Martinelli
Tenta série de Taylor na função f(x)=2^x e depois na função f(x)=ln(1+x). Essa última você avaliza ln(2) e na primeira expanda até a segunda derivada no máximo.

[obm-l] Mais uma questão legal

2005-12-05 Por tôpico Marcos Martinelli
Provar que (2005!)^(1/2005)=(2006!)^(1/2006).

[obm-l] Uma questão legal

2005-12-05 Por tôpico Marcos Martinelli
Artur, o limte procurado vale 1. Já tentei postar aqui a solução duas vezes masnão está entrando na lista. Usei uma desigualdade importante: n*ln(n)-n+1=ln(n!)=ln(n)+n*ln(n)-n+1 (para todo n natural), que pode ser verificada através de uma simples comparação de áreas com a integral da função

Re: [obm-l] Mais uma questão legal

2005-12-05 Por tôpico Marcos Martinelli
Interessante essa demonstração. Tinha pensado em algo mais complexo usando o fato de que n*ln(n)-n+1=ln(n!)=ln(n)+n*ln(n)-n+1, usar essa relação pra n e pra n+1 pra tentar forçar ln(n!)/n=ln[(n+1)!]/(n+1) e aí é so mostrar que a função f(x)=ln(x)/x+ln(x)+1/x é crescente, o que é imediato, basta

Re: [obm-l] Mais uma questão legal

2005-12-05 Por tôpico Marcos Martinelli
Usando a desigualdade do ln(n!), acredito que pode-se estimar melhor a desigualdade. O que você acha?

Re: [obm-l] Mais uma questão legal

2005-12-05 Por tôpico Marcos Martinelli
Seja f(x)=ln(x)/x-1/e para todo x real tal que x=e. f(e)=0 e f´(x)=(1-ln(x))/x^2=0 para todo x tal que x=e. Como pie = ln(pi)/pi-1/ef(e)=0 = ln(pi)/pi1/e = e*ln(pi)pi = ln(pi^e)ln(e^pi) = pi^ee^pi.

[obm-l] Uma questão legal

2005-12-02 Por tôpico Marcos Martinelli
Artur, consegui fazer sim. Tentei postar aqui o esboço da minha solução, mas acho que não entrou na lista. Basicamente utilizei o critério de comparação de áreas com a integral de uma função. É interessante notar também a seguinte relação: n*ln(n)-n+1=ln(n!)=ln(n)+n*ln(n)-n+1, para qualquer n

Re: [obm-l] soma

2005-11-30 Por tôpico Marcos Martinelli
Facilmente em termos não é? Se você utilizar esta abordagem existe um teorema que garante que a soma dos termos de uma p.a de grau 5 é um polinômio de grau 5 sem o termo independente. Porém terá que resolver um sistema 5x5 para encontrar seus coeficientes, o que dá trabalho. Sem levar em

Re: [obm-l] soma

2005-11-30 Por tôpico Marcos Martinelli
Não entendi. Você pode explicar melhor por favor. Obrigado! = Instruções para entrar na lista, sair da lista e usar a lista em http://www.mat.puc-rio.br/~nicolau/olimp/obm-l.html

Re: [obm-l] soma

2005-11-29 Por tôpico Marcos Martinelli
Esta soma é um polinômio de quinto grau em n, no caso n=40. Para conseguir desenvolvê-la você precisa conhecer a soma dos cubos e dos quadrados previamente. Existe uma solução longa mas interessante por combinatória. =

[obm-l] Mais uma bem legal!

2005-11-28 Por tôpico Marcos Martinelli
Provar que se A,B e C são matrizes reais comutativas de ordem 2 então temos det[(A+B+C)*(A^3+B^3+C^3-3*A*B*C)]=0. = Instruções para entrar na lista, sair da lista e usar a lista em

[obm-l] Uma questão legal

2005-11-26 Por tôpico Marcos Martinelli
Boa tarde pessoal. Achei essa questão super legal. Vou tentar produzi-la aqui: Calcular lim {ln(n!)/n-a(n)/n}, onde a seqüência {a_n} é definida da seguinte maneira: a(n)=Somatório(1=k=n){ln(k)*Somatório(k=j=n)[1/j]}. OBS: O limite é tomado quando n-+infinito.

[obm-l] Mais um Legal

2005-11-26 Por tôpico Marcos Martinelli
Sejam A,B e C ângulos de um triângulo ABC. Provar que 1/sen(A)+1/sen(b)=8/[3+2*cos(C)]. = Instruções para entrar na lista, sair da lista e usar a lista em http://www.mat.puc-rio.br/~nicolau/olimp/obm-l.html

Re: [obm-l] Mais um Legal

2005-11-26 Por tôpico Marcos Martinelli
Tenho uma legal sim. Utilizei a Desigualdade de Jensen na função f(x)=1/sen(x). Não é difícil verificar que a segunda derivada dessa função é positiva para todo x pertencente ao primeiro e segundo quadrantes. Temos então que: [1/sen(A)+1/sen(B)]/2=1/sen[(A+B)/2]=8/{2*[3+2*cos(C)]} =

Re: [obm-l] Problemas Selecionados de Matematica

2005-11-25 Por tôpico Marcos Martinelli
Não é difícil provar que existe m inteiro tal que a=m^4 e n inteiro tal que c=n^2. Basta decompor b e d em produto de fatores primos. Logo c-a=n^2-m^4=(n+m^2)*(n-m^2)=19=19*1=1*19 e então analisar os únicos casos válidos que se chega á resposta.

  1   2   >